Number theory

Prove that for n5n\geq5,(pn+1)3<p1×p2×p3pn(p_{n+1})^{3}<p_{1} \times p_{2}\times p_{3}\cdots p_{n} where pip_{i} is the ithi^\text{th} prime?

#NumberTheory

Note by Gnanananda Shreyas
10 months, 1 week ago

No vote yet
1 vote

  Easy Math Editor

This discussion board is a place to discuss our Daily Challenges and the math and science related to those challenges. Explanations are more than just a solution — they should explain the steps and thinking strategies that you used to obtain the solution. Comments should further the discussion of math and science.

When posting on Brilliant:

  • Use the emojis to react to an explanation, whether you're congratulating a job well done , or just really confused .
  • Ask specific questions about the challenge or the steps in somebody's explanation. Well-posed questions can add a lot to the discussion, but posting "I don't understand!" doesn't help anyone.
  • Try to contribute something new to the discussion, whether it is an extension, generalization or other idea related to the challenge.
  • Stay on topic — we're all here to learn more about math and science, not to hear about your favorite get-rich-quick scheme or current world events.

MarkdownAppears as
*italics* or _italics_ italics
**bold** or __bold__ bold

- bulleted
- list

  • bulleted
  • list

1. numbered
2. list

  1. numbered
  2. list
Note: you must add a full line of space before and after lists for them to show up correctly
paragraph 1

paragraph 2

paragraph 1

paragraph 2

[example link](https://brilliant.org)example link
> This is a quote
This is a quote
    # I indented these lines
    # 4 spaces, and now they show
    # up as a code block.

    print "hello world"
# I indented these lines
# 4 spaces, and now they show
# up as a code block.

print "hello world"
MathAppears as
Remember to wrap math in \( ... \) or \[ ... \] to ensure proper formatting.
2 \times 3 2×3 2 \times 3
2^{34} 234 2^{34}
a_{i-1} ai1 a_{i-1}
\frac{2}{3} 23 \frac{2}{3}
\sqrt{2} 2 \sqrt{2}
\sum_{i=1}^3 i=13 \sum_{i=1}^3
\sin \theta sinθ \sin \theta
\boxed{123} 123 \boxed{123}

Comments

\geq can be written inside Latex brackets to make \geq @Gnanananda Shreyas

A Former Brilliant Member - 10 months, 1 week ago

Note : Bertrand's Postulate states if n> 1 there is always at least one prime p such that n < p < 2n.

In other words, pk+1<2pk p_{k+1} < 2 p_{k}


so for n > 5 (pn+2)3<8(pn+1)3<8(p1×p2×p3....×pn)<pn+1×(p1×p2×p3....×pn) (p_{n+2})^{3} < 8 (p_{n+1})^3 < 8 ( p_1 \times p_2 \times p_3....\times p_n ) < p_{n+1} \times ( p_1 \times p_2 \times p_3....\times p_n ) as pi>8p_i > 8 for i>5i >5

Pop Wong - 10 months ago
×

Problem Loading...

Note Loading...

Set Loading...